LSAT and Law School Admissions Forum

Get expert LSAT preparation and law school admissions advice from PowerScore Test Preparation.

 Administrator
PowerScore Staff
  • PowerScore Staff
  • Posts: 8917
  • Joined: Feb 02, 2011
|
#33417
Complete Question Explanation

Justify the Conclusion. The correct answer choice is (C)

This is a challenging question, in part because the author used none of the usual premise or conclusion indicators to help us determine the structure of the argument. Nevertheless, you should realize that the entire stimulus supports the first sentence, because it explains why the proposed change to the patent system would have a chilling effect on research. The stimulus, when reworded, is structured as follows:
  • Premise: ..... Under current rules, researchers have one year after publication of a new discovery to patent the ..... ..... ..... ..... discovery, allowing research results to be shared.

    Premise: ..... The proposed change requires patent applications to precede initial publication, delaying
    ..... ..... ..... communication of discoveries.

    Cause ..... ..... ..... ..... ..... Effect

    Proposed change ..... :arrow: ..... Delay communication of discoveries

    Conclusion: ..... The proposed change is bound to have a chilling effect on scientific research.

    Cause ..... ..... ..... ..... ..... Effect

    Proposed change ..... :arrow: ..... Chilling effect on scientific research
The question stem asks us to identify a statement that, if assumed, would allow the conclusion to be properly drawn. Despite the word “assumed” in the stem, this is a Justify question because our job is not to identify a statement upon which the argument depends, but rather to prove the conclusion by adding a piece of information to the premises. The sufficient condition indicator (“if”) in the question stem is a reminder that you must select an answer that is sufficient to prove the conclusion by using the Justify Formula:
  • Premises + Answer choice = Conclusion
As with most Justify questions, there is a logical gap between the premises and the conclusion. The author argues the proposed change will have a chilling effect on scientific research, but the premises only suggest that it will delay the communication of discoveries. The correct answer choice must “connect the dots,” so to speak, by indicating that delays in the communication of discoveries will have a chilling effect on scientific research:
  • Cause ..... ..... ..... ..... ..... ..... ..... ..... Effect

    Delay communication of discoveries ..... :arrow: ..... Chilling effect on scientific research
This prephrase agrees with answer choice (C).

The correct answer choice can also be arrived at by the process of elimination. Given that the conclusion introduces a new, “rogue” element into the argument (“chilling effect”), the correct answer choice must connect that element to the rest of the argument. This eliminates all answer choices except for (C).

Answer choice (A): There is no reason to believe that encouraging more patent application will have a chilling effect on scientific research; even if it did, that would only strengthen the author’s conclusion, not prove it.

Answer choice (B): This answer choice strengthens the proposition that the current system has helped scientific research, but it does not establish that the proposed change would do the opposite.

Answer choice (C): This is the correct answer choice. If delays in communication of discoveries will have a chilling effect on scientific research, and the proposed change is bound to delay such communications, then it is reasonable to conclude that the change will have a chilling effect:
  • Cause ..... ..... ..... ..... Effect/Cause ..... ..... ..... ..... Effect

    Proposed ..... ..... ..... Delay communication ..... ..... Chilling effect
    change ..... :arrow: ..... ..... of discoveries ..... :arrow: ..... on scientific research
Answer choice (D): The fact that most researchers oppose the proposed change only strengthens the argument against it; it does not prove that the change will have a negative effect on scientific research.

Answer choice (E): How the current patent system facilitates scientific research has no bearing on the issue of whether the proposed change will have the opposite effect.
 htngo12
  • Posts: 40
  • Joined: May 19, 2016
|
#30219
Hi!

For this Justify question, I was able to pick the correct answer (A) as I was taking the test.

Now that I am going over just Justification Questions though the prep book, I am somewhat unsure how I was able to get to the answer. My current thought process is:

P1) Under current rules, initial publication of discovery ->1 full year after to patent -> research results are shared prior to patent.

P2) Under the proposed change, patent discovery-> publication of discovery-> research results are not shared.

Con: The proposed change to patent system will have effect on on scientific research.

I need an answer to justify the conclusion 100% true.

The gap that I see is that based on the current rule, it takes a full year to patent the discovery, while under the proposed the patent is prior, so for the conclusion to be true, I need an answer that will justify.

I guess I am having a hard time seeing the gap, even though I know it has to do with the difference in time frame.

For answer (A), states that the proposed change will encourage more patents to be filed. I not sure how to get to the answer.

Is there a way you walk me through the logic in this question?
 Emily Haney-Caron
PowerScore Staff
  • PowerScore Staff
  • Posts: 577
  • Joined: Jan 12, 2012
|
#30241
Hi htngo,

The correct answer on this one is actually C. I think that actually might be the problem you're having here! :) Just to make sure, though, let me walk you through the question:

Premise: The proposed change will delay the communication of discoveries.
Conclusion: The proposed change will have a chilling effect on research.

What's missing? Well, we need something connecting delayed communication and chilling effect. We get that with C.

I hope that helps!
 htngo12
  • Posts: 40
  • Joined: May 19, 2016
|
#33667
Yep! C) is the correct answer.
 BostonLawGuy
  • Posts: 52
  • Joined: Jul 13, 2018
|
#59932
Greetings, I understand the conditional logic and how, "delay communication :arrow: chills research," bridges the gap needed to arrive at the correct answer.

I wonder the difference between this and a necessary assumption? It appears the method we used to arrive at the correct answer is the same logical reasoning used when finding the necessary assumption.

In this case, is the necessary assumption also justify the conclusion?
 Robert Carroll
PowerScore Staff
  • PowerScore Staff
  • Posts: 1787
  • Joined: Dec 06, 2013
|
#61555
BLG,

A Justify answer choice and an Assumption answer choice can be the same thing. This is especially relevant when considering Supporter Assumptions. When you have a situation where it seems like a gap in the argument has to be closed, you prephrase some statement that will close the gap, then make sure of the following:

If it's a Justify question, make sure the answer choice is strong enough to completely close the gap. It can even be stronger than that!

If it's an Assumption question, make sure the answer choice isn't too strong - if it contains extraneous information, or covers more cases than are necessary for the argument to work, it may be wrong because it's too strong a statement.

I won't speculate on the counterfactual situation where this isn't a Justify question, but I'll say that sometimes a Supporter Assumption will have more qualified (weaker) language. Answer choice (C) is a strong statement, but, after all, that's good for a Justify, so that's good here!

Robert Carroll

Get the most out of your LSAT Prep Plus subscription.

Analyze and track your performance with our Testing and Analytics Package.